Difference between revisions of "Mock AIME 4 2006-2007 Problems/Problem 14"
(One intermediate revision by one other user not shown) | |||
Line 3: | Line 3: | ||
==Solution== | ==Solution== | ||
− | + | We will assume that there is at least one solution, otherwise the answer would be undefined. | |
+ | Using the binomial theorem it is obvious that <math>(577-k)^4 \equiv k^4 \pmod {577}</math>. Thus the solutions come in pairs <math>\{k,577-k\}</math>, and hence their average is <math>\dfrac{577}2 = 288.5</math>, and the answer is <math>\boxed{288}</math>. | ||
+ | |||
+ | (In this case, there are four solutions: <math>276</math>, <math>277</math>, <math>300</math>, and <math>301</math>.) | ||
+ | |||
+ | ---- | ||
+ | |||
+ | *[[Mock AIME 4 2006-2007 Problems/Problem 15| Next Problem]] | ||
+ | *[[Mock AIME 4 2006-2007 Problems/Problem 13| Previous Problem]] | ||
*[[Mock AIME 4 2006-2007 Problems]] | *[[Mock AIME 4 2006-2007 Problems]] |
Latest revision as of 00:52, 31 January 2009
Problem
Let be the arithmetic mean of all positive integers such that
.
Find the greatest integer less than or equal to .
Solution
We will assume that there is at least one solution, otherwise the answer would be undefined.
Using the binomial theorem it is obvious that . Thus the solutions come in pairs , and hence their average is , and the answer is .
(In this case, there are four solutions: , , , and .)